نتایح جستجو

  1. aras2213

    بله درسته!فکر کنم اسمش قضیه نیوتون هست. اصلا مگه پویا حرف اشتباه هم میزنه؟ d:

    بله درسته!فکر کنم اسمش قضیه نیوتون هست. اصلا مگه پویا حرف اشتباه هم میزنه؟ d:
  2. aras2213

    فکر کنم حل کردم ولی الان که میگی هنوز کسی جواب نداده یکم مشکوک شدم شاید جوب زدم

    فکر کنم حل کردم ولی الان که میگی هنوز کسی جواب نداده یکم مشکوک شدم شاید جوب زدم
  3. aras2213

    آره بابا بیچاره سوالای چندجمله ای رو هم من نمیذارم حل کنه:204:

    آره بابا بیچاره سوالای چندجمله ای رو هم من نمیذارم حل کنه:204:
  4. aras2213

    ماراتن هندسه

    پاسخ : ماراتن هندسه AoPS Forum - Not hard but cute (points lying on a circle) • Art of Problem Solving ---- دو نوشته به هم متصل شده است ---- http://www.artofproblemsolving.com/Forum/viewtopic.php?p=893744&sid=ca1271e8d3ac0b73d36507b 0718e0634#p893744 برعکس اینه که اثباتشون زیاد فرقی...
  5. aras2213

    ماراتن جبر (سطح ممتاز)

    پاسخ : ماراتن جبر (سطح ممتاز) برای قسمت 5 ، برای دومی هم که گفتید، با توجه به 5، بعد یه n از طرفین ساده میکنیم. ---- دو نوشته به هم متصل شده است ---- سوال بعد:همه n های طبیعی را بیابید که اعداد حقیقی، دو به دو متمایز و ناصفر موجود باشند که .
  6. aras2213

    آره یادم رفت بنویسم به تقلید از pco:4: من برا اون سوال نامساوی ایدم این بود که بگم مینیمم سمت...

    آره یادم رفت بنویسم به تقلید از pco:4: من برا اون سوال نامساوی ایدم این بود که بگم مینیمم سمت چپ برای z=0 اتفاق می افته ولی نتونستم
  7. aras2213

    رو سوال نظریه فکر نکردم رو نامساوی یکم فکر کردم که حل نشد چشم سوال میذارم

    رو سوال نظریه فکر نکردم رو نامساوی یکم فکر کردم که حل نشد چشم سوال میذارم
  8. aras2213

    ماراتن جبر (سطح ممتاز)

    پاسخ : ماراتن جبر (سطح ممتاز) f ثابت نیست. 1- --------------------------------------------------------------------------------------------------------------------------------------------------------------- فرض کنید (p(x,y بیان گر معادله باشد.در این صورت پس اگر (f(0 برابر با 0 نباشد تابع...
  9. aras2213

    بیخیال:4: خیلی داغونه:204:

    بیخیال:4: خیلی داغونه:204:
  10. aras2213

    سلام ممنون نه متاسفانه ندادم

    سلام ممنون نه متاسفانه ندادم
  11. aras2213

    آقا من الان با پویا حرف زدم یه جوب اساسی داشت:204: پویا ثابت کرده که وجود داره حلش هم درسته.

    آقا من الان با پویا حرف زدم یه جوب اساسی داشت:204: پویا ثابت کرده که وجود داره حلش هم درسته.
  12. aras2213

    سلام من گفتم n محدوده:4: نگفتم جواب نداره:4::4::4::4: حالا جوابت چیه؟:4:

    سلام من گفتم n محدوده:4: نگفتم جواب نداره:4::4::4::4: حالا جوابت چیه؟:4:
  13. aras2213

    کامل حل نکردم ولی فکر کنم ایده اصلیش این باشه: با توجه به این Faulhaber's formula - Wikipedia...

    کامل حل نکردم ولی فکر کنم ایده اصلیش این باشه: با توجه به این Faulhaber's formula - Wikipedia, the free encyclopedia میدونیم که [IMG] یه چند جمله ای درجه 11 بر حسب n هه.اسم این چند جمله ای رو f میذاریم .حالا یه مجموعه ای وجود داره که جمع اعضاش حداقل [IMG] هست.پس [IMG] که فقط برای تعداد...
  14. aras2213

    سلامسوال چیه؟ برا کجاست؟

    سلامسوال چیه؟ برا کجاست؟
  15. aras2213

    سلام ببخشید برای یه مدت سرعت اینترنت خیلی پایین بود و اگه میخواستم چیزی بنویسم خیلی طول میکشید...

    سلام ببخشید برای یه مدت سرعت اینترنت خیلی پایین بود و اگه میخواستم چیزی بنویسم خیلی طول میکشید. خوب اول در مورد اون نماد ،خودت میتونی برا خودت استفاده کنی ولی من جایی ندیدم که از این استفاده کنن(خیلی هم زیاد چیزی در این باره نخوندم(کل اطلاعاتی که در مورد این دارم 3 سوال و یک صفحه درسه)که بخوام...
  16. aras2213

    ماراتن نامساوي

    پاسخ : ماراتن نامساوي که با استفاده از نامساوی کوشی کافیه ثابت کنیم: که با توجه به فرض مساله و نامساوی شور درست است.
  17. aras2213

    ماراتن جبر (سطح پیشرفته)

    پاسخ : ماراتن جبر (سطح پیشرفته) سوال خوبیه:4:(من هم اولین بار که دیدم اینا رو نفهمیدم، بعد از استاد شاولی پرسیدم توضیح دادند برام) همون که ضرایبش به پیمانه 2 حساب میشن.مثلا اگر ، اون وقت برابر با میشه.حالا خاصیت مهم این کاری که ما انجام میدیم اینه که اگه معادله برقرار باشه اون وقت اگر دو...
  18. aras2213

    ماراتن هندسه

    پاسخ : ماراتن هندسه 3 دایره ی دو به دو به یکدیگر مماس خارجند.مماس مشترک داخلی (خط ) دایره ی را برای بار دوم در P قطع میکند.(یعنی از دو محل تقاطع های با ، P از دور تره.)مماس مشترک خارجی دایره ی را در قطع میکند.نشان دهید نیمساز زاویه ی است.
  19. aras2213

    ماراتن جبر (سطح پیشرفته)

    پاسخ : ماراتن جبر (سطح پیشرفته) فرض کنید که .حالا این معادله رو به پیمانه 2 نگاه کنید.چون که پس بدست میاد که .حالا چون عبارت داخل پرانتز تحویل ناپذیره، . در نتیجه . پس .حالا اگه ،.اما P,Q دارای ضرایبی صحیح هستند پس به شکل هست ولی 1/2 این جوری نیست.
  20. aras2213

    ماراتن نظریه ی اعداد (سطح ممتاز)

    پاسخ : ماراتن نظریه ی اعداد (سطح ممتاز) سوال بعد:همه چند جمله ای های با ضرایب صحیح را بیابید که برای هر ( :4:...:4: )داشته باشیم .
بالا